Ob Finals Williams Samplex
Ob Finals Williams Samplex
Ob Finals Williams Samplex
Delivery
CHAPTER 27
Vaginal Delivery
27–1. The most common position for vaginal delivery is 27–6. All except which of the following statements
which of the following? regarding delivery of the fetal body are true?
a. Squatting a. Usually delivers without delay after the shoulders.
b. Knee-chest b. Hooking the axillae can be employed to hasten
c. Dorsal lithotomy delivery of the body.
d. None of the above c. Moderate outward traction on the head may
be used to help delivery.
27–2. At the time of perineal distention prior to vaginal d. Immediate nasopharyngeal suction should be
delivery, which of the following is correct? performed after delivery.
a. Most presentations are occiput posterior.
27–7. What are the benefits of delayed cord clamping in
b. Infrequently the vertex may be occiput anterior.
preterm infants?
c. The encirclement of the largest diameter by the
a. Decrease IVH
vulvar ring is called crowning.
b. Decrease NEC
d. All of the above
c. Decrease need for blood transfusion
27–3. Which of the following maneuvers may protect the d. All of the above
perineum during delivery?
a. Slow delivery of the head 27–8. A 34-year-old primigravida at 41 weeks’ gestation
presents for induction of labor. She tells you she
b. One-hand support of the perineum
would like you to perform delayed cord clamping.
c. Guidance to deliver the smallest head diameter What risks are there to delayed cord clamping in the
through the introitus term infant?
d. All of the above a. Lower Apgar scores
b. Postpartum hemorrhage
27–4. Which of the following is true regarding nuchal
cords? c. Increased hyperbilirubinemia
a. Found in 25% of deliveries at term d. Respiratory distress syndrome from polycythemia
b. Are more common in preterm infants
27–9. All except with of the following is true regarding
c. Tight nuchal cords are associated with 10% of all occiput transverse position of the fetal head in the
deliveries maternal pelvis?
d. Presence of a nuchal cord is associated with worse a. Seen with android pelvises
neonatal outcomes
b. Easiest way to rotate is manually
27–5. Following delivery of the fetal head, which of the c. Can be rotated with Kielland forceps
following occurs? d. Will rotate to occiput anterior with hypotonic
a. The head rotates anterior contractions
b. The head undergoes internal rotation
c. The head undergoes external extension
d. The head rotates into the transverse position
27–10. Which of the following is true regarding persistent 27–15. A 33-year-old multigravida is in clinic at 39 weeks’
occiput posterior (OP) position in labor? gestation and believes she has big baby. An ultra-
a. Seen more often in multiparous women sound is performed, and the estimated fetal weight is
CHAPTER 27
9 pounds. Which of the following is true?
b. 2–10% of cephalic fetuses deliver in the OP
position a. Planned cesarean delivery may be offered.
c. Gynecoid pelvises predispose to persistent OP b. Elective induction should be scheduled soon.
position c. Shoulder dystocia may be accurately predicted.
d. Epidural analgesia is not a risk for persistent OP d. Fetal macrosomia is a risk factor for shoulder
position dystocia.
27–11. For women who deliver vaginally with persistent 27–16. The following illustration demonstrates which
occiput posterior (OP) position, which of the maneuver for reduction of shoulder dystocia?
following is true?
a. Transabdominal ultrasound can aid in identifying
the OP position.
b. There is increased blood loss compared to occiput
anterior position.
c. There are more third- and fourth-degree
lacerations compared to occiput anterior position.
d. All of the above
27–18. Which of the following statistics concerning home 27–24. Approximately 2% of singleton births are accompanied
birth is accurate? by a delayed third stage. Possible reasons for this
a. 0.7% of deliveries in the United States are include which of the following?
a. Placenta adherens
SECTION 8
27–20. The World Health Organization classifies genital 27–26. Risk factors for obstetrical anal sphincter injuries
mutilation into four types. Which of the following include which of the following?
descriptions is true? a. Multiparity
a. Type I: Partial or total removal of the clitoris b. Mediolateral episiotomy
and/or prepuce c. Left occiput anterior position
b. Type II: Partial or total removal of the clitoris d. None of the above
and the labia minora
c. Type III: Partial or total removal of the labia 27–27. What is true concerning the illustrated perineal
minora and/or majora and infibulation without episiotomy below?
or without clitoridectomy
d. All of the above
27–28. Indications for episiotomy include which of the 27–31. After completion of a fourth-degree perineal
following? laceration repair, you order which of the following
a. Breech delivery medications?
CHAPTER 27
b. Shoulder dystocia a. Zosyn
c. Persistent occiput posterior position b. Ampicillin
d. All of the above c. Dinoprostone
d. Second-generation cephalosporin
27–29. Which of the following is true regarding the repair
of third-degree lacerations? 27–32. A 23-year-old G1P1 without medical problems has
a. Antibiotics are not necessary. just undergone a spontaneous vaginal delivery. After
delivery of the placenta, you note continued brisk
b. The preferred suture is chromic.
bleeding. Exam suggests uterine atony. Which of the
c. The overlapping technique is only appropriate for following is the next most appropriate medication?
type 3C lacerations.
a. Dinoprostone
d. The strength of the closure is derived from the
striated muscle of the sphincter. b. Rectal misoprostol
c. Intravenous magnesium sulfate
27–30. A 25-year-old G1P1 calls your office on postpartum d. Intramuscular methylergonovine
day 6 complaining of increasing perineal pain
for the last 24 hours. She underwent a successful
forceps-assisted vaginal delivery complicated by a
third-degree laceration. Which of the following is
part of the differential diagnosis?
a. Wound infection
b. Urinary retention
c. Uterine involution
d. Excessive ambulation
CHAPTER 28
Breech Presentation
Reproduced with permission from Cunningham FG, Leveno KJ, Bloom SL, et al (eds): Breech Reproduced with permission from Cunningham FG, Leveno KJ, Bloom SL, et al (eds): Breech
delivery. In Williams Obstetrics, 25th ed. New York, McGraw-Hill, 2018, Figure 28-1. delivery. In Williams Obstetrics, 25th ed. New York, McGraw-Hill, 2018, Figure 28-3.
Reproduced with permission from Cunningham FG, Leveno KJ, Bloom SL, et al (eds): Breech
delivery. In Williams Obstetrics, 25th ed. New York, McGraw-Hill, 2018, Figure 28-2.
28–2. Among singleton pregnancies, breech presentation 28–7. A 29-year-old multigravida is admitted to your
persists in what percentage at term? antepartum service after premature rupture of
a. 1–2% membranes at 22 weeks’ gestation. She is now
22 weeks 3 days and, after consultation with
SECTION 8
b. 3–5%
neonatology, she desires full resuscitation options
c. 6–7% at 23 weeks’ gestation. Her fetus is breech. She
d. 8–9% understands that many factors are involved but
inquires about which mode of delivery she should
28–3. Risk factors for breech presentation include which of anticipate. Which of the following statements is
the following? most appropriate?
a. Nulliparity a. Cesarean would be considered after 23 weeks’
b. Prior breech fetus gestation.
c. Singleton gestation b. Cesarean would be recommended after 25 weeks’
gestation.
d. None of the above
c. Vaginal delivery would be recommended until
28–4. A 24-year-old primigravida presents to labor and 23 weeks’ gestation.
delivery at 38 weeks’ gestation. She is contracting d. All of the above
regularly and on exam she is 4 cm dilated, 25%
effaced, –3 station, and the fetal presenting part 28–8. Which of the following is not true regarding maternal
is suspected to be a foot. What is the next most morbidity and mortality in breech delivery?
appropriate step in her management? a. Maternal death is less likely if the breech fetus is
a. Ultrasound for confirmation of fetal presentation delivered via cesarean.
b. Anesthesia consultation for pain management b. Hysterotomy extensions can occur with the use
during labor of forceps during cesarean delivery of the breech
c. Leopold maneuvers for further characterization of fetus.
fetal position c. Anesthesia given to aid in uterine relaxation for
d. Preparation for cesarean delivery due to non-cephalic vaginal delivery can increase the risk for postpartum
presentation in labor hemorrhage.
d. Genital tract lacerations can occur with both
28–5. The Term Breech Trial has heavily influenced mode vaginal and cesarean delivery of the breech fetus,
of delivery since its publication. Which of the and the risk of infection is increased.
following is a criticism of the study?
a. Mode of delivery was not randomized. 28–9. A 28-year-old G3P2 presented in advanced labor
with a fetus in frank breech presentation, and
b. More than 10% of study participants had
she delivered vaginally. Her 2-year-old is now
radiologic pelvimetry, which may have influenced
undergoing therapy for hip dysplasia and Erb
management.
palsy. Which of the following is true regarding her
c. Many of the outcomes included in the “serious daughter’s condition?
neonatal morbidity” composite did not portend a. Hip dysplasia is a known risk of vaginal breech
long-term infant disability. delivery due to physical trauma.
d. All of the above b. Since it was not a shoulder dystocia, the Erb palsy
cannot be attributed to the delivery.
28–6. Which of the following best characterizes the state
of the literature regarding vaginal delivery of a term c. Both the hip dysplasia and Erb palsy could have
been avoided if she had a cesarean delivery.
singleton breech?
d. Hip dysplasia is seen more frequently after
a. Literature is mixed
breech presentation and is unaffected by mode of
b. Literature clearly shows trend to harm with delivery.
planned vaginal delivery
c. Literature clearly shows no extra harm with
planned vaginal delivery
d. None of the above
28–10. When assessing candidacy for planned vaginal breech 28–13. During a partial breech extraction, in the step
delivery, why are maternal pelvimetry and fetal pictured below, assisted delivery of the legs involves
ultrasound biometry correlation important? which of the following?
CHAPTER 28
a. The head is often larger than the breech.
b. The head of a breech fetus does not undergo
molding during labor.
c. Ultrasound will help identify fetal anomalies that
would alter candidacy.
d. All of the above
28–14. The image shown below demonstrates a fetus in 28–16. To resolve the complication shown in the image
which position? below, which of the following maneuvers should not
be employed?
SECTION 8
Reproduced with permission from Cunningham FG, Leveno KJ, Bloom SL, et al (eds): Breech
delivery. In Williams Obstetrics, 25th ed. New York, McGraw-Hill, 2018, Figure 28-4.
a. Sacrum posterior Reproduced with permission from Yeomans ER: Vaginal breech delivery. In Yeomans ER,
Hoffman BL, Gilstrap LC III, et al (eds): Cunningham and Gilstrap’s Operative Obstetrics,
b. Left sacrum anterior 3rd ed. New York, McGraw-Hill, 2017, Figure 31-13.
c. Right sacrum anterior
d. Left sacrum transverse a. The fetus should be pulled downward to release
the arm.
28–15. Which of the following statements is false regarding b. The fetus should be rotated 180 degrees clockwise
the tenets of partial breech extraction? in the case of the left arm.
a. The breech is allowed to deliver spontaneously to c. The fetus should be rotated 180 degrees counter-
the level of the umbilicus. clockwise for the right arm.
b. Following delivery of the legs, the fetal bony d. The fetus is gently pushed upward back into the
pelvis is grasped with both hands with fingers pelvis for second attempt at rotation if a primary
resting on the anterior superior iliac crests and maneuver is unsuccessful.
thumbs on the sacrum.
c. After delivery of the fetal legs, steady, gentle, down- 28–17. Which of the following correctly describes aspects
ward traction is employed until the lower halves of of the Mariceau maneuver for delivery of the
the scapulae are delivered with no effort to deliver aftercoming head?
shoulders and arms until one axilla is seen. a. Suprapubic pressure is applied by an assistant to
d. Once the breech has delivered to the level of the aid in keeping the head extended.
umbilicus, request cessation of maternal expulsive b. Two fingers of one hand grasp the shoulders of
effort and patiently await spontaneous delivery the back-down fetus from below while the other
of the shoulders with gentle support of the lower hand grasps both fetal feet, lifting up and over the
portion of the fetal body. maternal abdomen.
c. The index and middle finger of one hand are
applied over the maxilla to flex the head while
the fetal body rests on the palm of the same hand
with legs straddling the forearm.
d. Once the suboccipital region of the fetal neck
appears under the maternal symphysis, the fetal
body is lowered below the plane of the maternal
perineum to accomplish flexion and subsequent
delivery of the head.
28–18. What is the utility of the maneuver demonstrated in 28–19. Which of the following is true regarding the
the image below? procedure being demonstrated in this image?
CHAPTER 28
Reproduced with permission from Cunningham FG, Leveno KJ, Bloom SL, et al (eds): Breech
delivery. In Williams Obstetrics, 25th ed. New York, McGraw-Hill, 2018, Figure 28-11c.
28-21. The patient in Question 28–20 was determined 28–24. What is the name of the process by which a frank
to have two feet and a loop of umbilical cord at breech fetus is manually converted to a footling
the vaginal introitus. Once in the operating room, breech presentation within the uterus during a
general anesthesia is emergently induced and the cesarean or vaginal delivery?
SECTION 8
28–29. Which of the following is an absolute contraindica- 28–32. Internal podalic version is best characterized in
tion to external cephalic version? which of the following?
a. Early labor a. Manipulation within the uterus to yield a breech
CHAPTER 28
b. Twin gestation presentation, typically reserved for delivery of a
second twin
c. Oligohydramnios
b. Manipulation within the uterus to yield a vertex
d. Fetal-growth restriction
presentation, typically reserved for the delivery of
a second twin
28–30. Which of the following is true regarding perfor-
mance of external cephalic version? c. Manipulation within the uterus to yield a vertex
presentation for a preterm breech fetus immedi-
a. Immediate induction is indicated if version is
ately after rupture of membranes
successful.
d. None of the above
b. Anti-D immune globulin is administered to
Rh-D negative women. 28–33. A patient presents for her postpartum visit after
c. Increased amnionic fluid is correlated with suc- undergoing cesarean delivery at 40 weeks’ gestation
cess, and amnioinfusion is routinely performed to for breech presentation. No uterine anomaly was
increase version success. identified during the surgery. Is breech presentation
d. All of the above more likely with her next pregnancy?
a. Yes, with one prior breech presentation at term,
28–31. External cephalic version success is improved with recurrence is 6%.
which of the following? b. Yes, with one prior breech presentation at term,
a. Acute tocolysis recurrence is 10%.
b. Intravenous sedation c. Yes, with one prior breech presentation at term,
c. Amnioinfusion to augment amnionic fluid recurrence is 28%.
volume d. No, in the absence of a uterine anomaly, fetal
d. All of the above presentation is random, and recurrence is not
increased above baseline risk.
CHAPTER 29
29–1. What is the ratio of vacuum-assisted to forceps- 29–6. Which of the following criteria must be met prior to
assisted vaginal deliveries? performing an operative vaginal delivery?
a. 1:1 a. Adequate anesthesia
b. 2:1 b. Maternal bladder emptied
c. 3:1 c. Known fetal head position
d. 4:1 d. All of the above
29–2. All except which of the following are acceptable 29–7. A 32-year-old multigravida with an epidural in place
indications for operative vaginal delivery? has been pushing for 2 hours with poor descent.
a. Aortic stenosis The fetal head is positioned left occiput posterior.
Forceps delivery of this patient would be classified as
b. Maternal exhaustion
which of the following?
c. Pelvic floor protection
a. Low
d. 2nd stage labor ≥2 hours in a multipara with an
b. High
epidural
c. Outlet
29–3. A 31-year-old primigravida undergoing induction of d. Low outlet
labor reaches the 2nd stage of labor after 36 hours.
Before beginning to push she says she is too tired 29–8. Which of the following describes forceps that are
and desires an operative vaginal delivery. Which of applied to the fetal head with the scalp visible at the
the following precludes her from having an elective introitus without separation of the labia?
operative vaginal delivery? a. Low
a. Head is at +1 station b. Mid
b. Head is in occiput anterior position c. High
c. Scalp is visible at the introitus without labial d. Outlet
separation
d. All of the above 29–9. Maternal morbidity with forceps delivery is most
closely predicted by which of the following?
29–4. Which of the following is a prerequisite for vacuum a. Fetal station
extraction, but not a forceps-assisted vaginal delivery?
b. Maternal parity
a. Epidural anesthesia
c. Fetal gestational age
b. Minimum +2 station
d. Length of the second stage
c. Assistant to create suction
d. Minimum 34 weeks’ gestation 29–10. In addition to operative vaginal delivery, which of
the following is a risk factor for urinary retention?
29–5. Which of the following is true of high forceps? a. Parity
a. No role in modern obstetrics b. Episiotomy
b. Indicated for fetal bradycardia c. Labial swelling
c. Forceps applied when the fetal head is engaged d. Length of the second stage
d. Indicated for those with a prolonged second stage
29–11. Which of the following occurs more frequently 29–15. A subgaleal hemorrhage is seen more frequently with
with forceps-assisted vaginal delivery compared to which of the following delivery routes?
vacuum-assisted vaginal deliveries? a. Forceps delivery
a. Uterine atony
SECTION 8
b. Cesarean delivery
b. Urinary retention c. Vacuum extraction
c. Prolonged hospital stay d. Spontaneous vaginal delivery
d. Vaginal wall lacerations
29–16. A 32-year-old primigravida with an epidural has
29–12. Which of the following interventions may reduce been pushing for 3 hours. Fetal position is occiput
maternal perineal laceration during a forceps-assisted posterior, station is +2, and estimated fetal weight
vaginal delivery? is 3400 grams. You consider an operative vaginal
a. Early disarticulation delivery for maternal exhaustion. Which of the
following is associated with failure of an operative
b. Mediolateral episiotomy
delivery?
c. Cessation of pushing during disarticulation
a. Primigravida
d. All of the above
b. Full maternal bladder
29–13. The perinatal complication shown below is seen c. Occiput posterior position
more frequently with which of the following delivery d. Estimated fetal weight ≥3400 grams
routes?
29–17. The opening in this forceps blade mainly serves
which of the following functions?
a. Forceps delivery
b. Cesarean delivery
a. Protects the fetal ears
c. Vacuum extraction
b. Allows blades to grip the fetal head firmly
d. Spontaneous vaginal delivery
c. Offers a smaller metal surface area against the
29–14. What is the mechanism for the injury pictured in fetal skull
Question 29–13? d. Provides diminished traction forces against the
a. Intracranial hemorrhage maternal vaginal sidewall
b. Shoulder dystocia from a forceps delivery
29–18. During placement of a forceps blade, what is the
c. Compression of the facial nerve by the forceps purpose of having the right hand between the fetal
blade head and the vaginal sidewall?
d. None of the above a. Protects the fetal ears
b. Identifies the ischial spines
c. Reduces maternal discomfort during placement
d. Guides the blade into position and protects the
vaginal sidewall
29–19. In the setting of an occiput posterior position, 29–24. In attempting a rotation from occiput anterior to
correctly placed blades are equidistant from what occiput posterior, which type of forceps is best
landmark? employed?
CHAPTER 29
a. Fetal ears a. Piper
b. Sagittal suture b. Luikart
c. Lambdoidal sutures c. Kielland
d. Midline of the face and brow d. Simpson
29–20. How is asynclitism resolved after placement of 29–25. In all except which of the following presentations
forceps? may forceps be used?
a. It cannot be resolved a. Mentum anterior
b. Removing and replacing the forceps b. Occiput posterior
c. Pulling and/or pushing each branch along the c. Mentum posterior
long axis d. Occiput transverse
d. Proceeding with downward traction, and it will
spontaneously resolve 29–26. Which of the following is true concerning the
vacuum device pictured below compared to a soft
29–21. What forces are produced by a forceps-assisted cup system?
vaginal delivery?
a. Friction
b. Traction
c. Compression
d. All of the above
29–23. Which of the following pelvic types is generally a. Allows easier placement
associated with persistent occiput posterior position? b. Allows more traction force
a. Android c. Higher scalp laceration rates
b. Gynecoid d. All of the above
c. Anthropoid
d. Platypelloid
29–27. In the diagram below, which letter corresponds to 29–29. Which of the following is the preferred total negative
the flexion point where the center of the cup should pressure generated prior to initiation of traction
be placed? during vacuum extraction?
a. 0.2 kg/cm2
SECTION 8
b. 0.8 kg/cm2
c. 1.2 kg/cm2
d. 1.6 kg/cm2
CHAPTER 29
number answer cited Header cited
29–1 d p. 553 Introduction
29–2 c p. 553 Indications
29–3 a p. 553 Indications
29–4 d p. 554 Classification and Prerequisites
29–5 a p. 554 Classification and Prerequisites
29–6 d p. 554 Table 29-1
29–7 d p. 554 Table 29-1
29–8 d p. 554 Table 29-1
29–9 a p. 553 Classification and Prerequisites
29–10 b p. 554 Classification and Prerequisites
29–11 d p. 555 Lacerations
29–12 d p. 555 Lacerations
29–13 a p. 555 Acute Perinatal Injury
29–14 c p. 556 Mechanisms of Acute Injury
29–15 c p. 555 Acute Perinatal Injury
29–16 c p. 556 Trial of Operative Vaginal Delivery
29–17 b p. 557 Design
29–18 d p. 557 Blade Application and Delivery
29–19 d p. 558 Blade Application and Delivery
29–20 c p. 558 Blade Application and Delivery
29–21 d p. 560 Blade Application and Delivery
29–22 a p. 561 Blade Application and Delivery
29–23 c p. 561 Blade Application and Delivery
29–24 a p. 561 Blade Application and Delivery
29–25 c p. 562 Face Presentations
29–26 d p. 562 Vacuum Extractor Design
29–27 a p. 563 Technique
29–28 d p. 563 Technique
29–29 b p. 563 Technique
29–30 a p. 564 Technique
29–31 d p. 554 Table 29-1
29–32 b p. 564 Technique
CHAPTER 30
30–1. Of the following indications for primary cesarean 30–6. A 22-year-old G1 at 39 weeks’ gestation with no
delivery, which is least common? prenatal care arrives on labor and delivery in active
a. Fetal jeopardy labor. When completing her delivery consents she
declines all blood products due to religious beliefs.
b. Labor dystocia
Which of the following interventions would be most
c. Placenta previa helpful at this time?
d. Malpresentation a. Minimize blood draws
b. Erythropoietin administration
30–2. Which of the following indications for primary
cesarean delivery is most inappropriate? c. Iron and folic acid administration
a. Malpresentation d. Proactive administration of uterotonics if atony
occurs
b. Prior cesarean delivery
c. Unknown uterine scar type 30–7. A 20-year-old primigravida presents at 40 weeks’
d. Early-onset severe preeclampsia gestation with elevated blood pressures and a headache.
She is diagnosed with severe preeclampsia, and
30–3. Which of the following is least likely to have induction of labor is begun. After 28 hours she has
contributed to the rising rate of primary cesarean made no cervical change and the decision is made
delivery between 1970 and 2009? to proceed with primary cesarean delivery. Which
a. Abnormal placentation of the following interventions has been proven to
decrease her risk of postoperative wound infection,
b. Electronic fetal monitoring
in addition to 2 grams of cefazolin prior to skin
c. Decrease in rates of vaginal birth after cesarean incision?
delivery
a. Shaving of surgical site
d. Decrease in vaginal delivery rates for breech
b. Azithromycin 500 mg intravenous
presentation
c. Preparation of the vagina with povidone-iodine
30–4. A 34-year-old primigravida at 39 weeks’ gestation d. All of the above
undergoes primary cesarean delivery on maternal
request. Compared to a woman with a vaginal 30–8. A 29-year-old woman at 40 weeks’ gestation is
delivery, which of the following adverse neonatal scheduled to undergo primary cesarean delivery for
outcomes is more likely to occur? malpresentation. She weighs 310 pounds, with a
a. Infection body mass index of 54 kg/m2. She asks which type of
skin incision is recommended, and you recommend
b. Birth trauma
a periumbilical vertical midline incision. Which of
c. Respiratory distress syndrome the following is true regarding a vertical midline as
d. Hypoxic ischemic encephalopathy compared to transverse skin incision for this patient?
a. Decreased risk of neuropathy
30–5. Which of the following maternal obstetrical
b. Greater ease with wound care
complications is increased with cesarean delivery
as compared to vaginal delivery? c. Decreased risk of subfascial hematoma
a. Infection d. All of the above
b. Hemorrhage
c. Thromboembolism
d. All of the above
30–9. A 42-year-old G3P2 is scheduled to undergo a 30–13. A 30-year-old G4P3 at 39 weeks’ gestation under-
repeat cesarean delivery. She reports heavy menstrual goes repeat cesarean delivery, which is remarkable
bleeding outside of pregnancy and requests cesarean for extensive adhesive disease. Postoperatively she is
CHAPTER 30
hysterectomy. You inform the patient this is not noted to have minimal urine output, abdominal
an indication for cesarean hysterectomy given the distention, and severe abdominal pain. Her vital
associated risks. You counsel her that cesarean signs are remarkable for mild tachycardia with blood
hysterectomy increases her risk of which of the pressure 118/78 mmHg, and her hematocrit is noted
following? to be stable from the preoperative value of 32%. Her
a. Infection Foley is replaced and she receives multiple boluses
of crystalloid without improvement in urine output.
b. Chronic pelvic pain
What is the next best step in management?
c. Urinary tract damage
a. Transfusion
d. None of the above
b. Lasix administration
30–10. The patient in Question 30–9 is completing the c. Intravenous pyelography
consent for her repeat cesarean delivery. You discuss d. Abdominal computed tomography with
the risks of bowel, bladder, and ureteral injury. Which cystography
of the following most closely approximates the risk of
ureteral injury at the time of cesarean delivery? 30–14. A 30-year-old G3P3 presents to the emergency
a. 1 in 500 department on postoperative day 4 complaining of
24 hours of severe abdominal pain and intractable
b. 1 in 1000
nausea and vomiting. Her oral temperature is 39 ºC;
c. 1 in 2000 she is tachycardic with a heart rate of 140 bpm and
d. 1 in 3000 appears pale and diaphoretic. What is the best next
step in management?
30–11. A 25-year-old G2P1 at 39 weeks’ gestation is a. Intravenous antibiotics
undergoing trial of labor after a cesarean delivery
b. Exploratory laparotomy
in her previous pregnancy. She progresses to the
second stage of labor, but after 3 hours of maternal c. Placement of nasogastric tube, bowel rest
effort a fetal bradycardia occurs for which an d. None of the above
emergent cesarean delivery is performed. Which
of the following circumstances increases her risk of 30–15. Which of the following is a risk factor for urinary
unintentional cystotomy? retention after cesarean delivery?
a. History of prior cesarean delivery a. Hemorrhage
b. Need for emergent cesarean delivery b. Prior cesarean delivery
c. Cesarean delivery in the second stage of labor c. Postoperative narcotic analgesia
d. All of the above d. All of the above
30–16. A 21-year-old primigravida at 41 weeks’ gestation 30–17. Although not recommended by the American
is undergoing labor induction for oligohydramnios. College of Obstetricians and Gynecologists, elective
She progresses to the second stage of labor, but the cesarean delivery on maternal request should only be
fetal head does not descend below 0 station despite considered as an option when which of the following
SECTION 8
30–22. Compared with a vertical midline incision, a 30–25. Failure to recognize dextrorotation of the uterus
Pfannenstiel incision offers which of the following prior to hysterotomy increases the risk of damage to
benefits? which structure?
CHAPTER 30
a. Less postoperative pain a. Left ureter
b. Improved cosmetic result b. Right ureter
c. Less risk of incisional hernia c. Left uterine artery
d. All of the above d. Right uterine artery
30–23. When performing dissection through a Pfannenstiel 30–26. Extension of the hysterotomy may be accomplished
incision, the two fascial layers are incised as with two different methods, as shown below.
illustrated in this image. The first layer encountered, Compared with blunt extension, the use of bandage
which is incised in this image, is the aponeurosis of scissors for sharp extension has been associated with
what muscle? an increase in which of the following?
Amnionic
sac
Reproduced with permission from Cunningham FG, Leveno KJ, Bloom SL, et al (eds):
Cesarean delivery and peripartum hysterectomy. In Williams Obstetrics, 25th ed. New
York, McGraw-Hill, 2018, Figure 30-5.
Reproduced with permission from Word L, Hoffman BL: Surgeries for benign gynecologic
conditions. In Hoffman BL, Schorge JO, Schaffer JI, et al (eds) Williams Gynecology, 2nd
ed. New York, McGraw-Hill, 2012, Figure 41-2.1. a. Blood loss
b. Operative time
a. Transversalis c. Unintended extensions
b. Internal oblique d. All of the above
c. External oblique
d. Transversus abdominis
30–27. During cesarean delivery a hysterotomy is made in 30–28. Compared with manual extraction, spontaneous
the lower uterine segment, as shown here. In which delivery of the placenta with fundal massage, as
of the following settings should the incision be made shown below, has been shown to reduce the risk of
relatively higher on the uterus to avoid uterine vessel which complication?
SECTION 8
Uterine incision
Myometrium
Placenta
Reproduced with permission from Cunningham FG, Leveno KJ, Bloom SL, et al (eds):
Reproduced with permission from Cunningham FG, Leveno KJ, Bloom SL, et al (eds):
Cesarean delivery and peripartum hysterectomy. In Williams Obstetrics, 25th ed. New
Cesarean delivery and peripartum hysterectomy. In Williams Obstetrics, 25th ed. New
York, McGraw-Hill, 2018, Figure 30-9.
York, McGraw-Hill, 2018, Figure 30-4.
30–29. Which of the following is a disadvantage of uterine 30–32. Which of the following is a theoretical benefit of
exteriorization for repair of the hysterotomy, as show using a monofilament suture such as polydioxanone
below? (PDS) to close the fascial layer, as opposed to a
CHAPTER 30
braided suture such as polyglactin (vicryl)?
a. Decreased risk of infection
b. Lower half-life tensile strength
c. Decreased risk of knot slippage
d. All of the above
Reproduced with permission from Cunningham FG, Leveno KJ, Bloom SL, et al (eds):
Cesarean delivery and peripartum hysterectomy. In Williams Obstetrics, 25th ed. New
Reproduced with permission from Cunningham FG, Leveno KJ, Bloom SL, et al (eds):
York, McGraw-Hill, 2018, Figure 30-10.
Cesarean delivery and peripartum hysterectomy. In Williams Obstetrics, 25th ed. New
York, McGraw-Hill, 2018, Figure 30-12.
a. Increased blood loss
b. Increased febrile morbidity a. Maternal morbid obesity
c. Increased nausea and vomiting b. Densely adherent bladder
d. Increased postoperative infection c. Back-down transverse fetal lie
d. All of the above
30–30. Which of the following interventions is most likely
to decrease adhesion formation at the time of 30–34. A 33-year-old G3P2 with two prior cesarean
cesarean delivery? deliveries presents at 35 weeks’ gestation with active
a. Achieving hemostasis vaginal bleeding and fetal heart rate decelerations.
b. Closure of the peritoneum She is taken for emergent repeat cesarean delivery
and is found to have a placenta previa with accreta
c. Placement of an adhesion barrier at the that requires cesarean hysterectomy. Compared
hysterotomy with patients who have this procedure planned, this
d. All of the above woman is at increased risk for which of the following
complications?
30–31. What is a potential advantage of closure of the a. Bladder injury
parietal peritoneum prior to closure of the fascia at
the time of cesarean delivery? b. Ureteral injury
a. Shorter operative time c. Blood transfusion
b. Less adhesion formation d. All of the above
c. Decreased postoperative pain 30–35. Women who have normal blood volume expansion
d. Avoidance of distended bowel at the time of in pregnancy and a hematocrit of at least 30%
fascial closure will usually tolerate blood loss up to what volume
without hemodynamic compromise?
a. 2000 mL
b. 3000 mL
c. 4000 mL
d. 5000 mL
CHAPTER 31
31–1. Which of the following factors favors a successful 31–4. You are taking care of a 25-year-old G2P1 at
trial of labor in a woman with a prior cesarean delivery? 36 weeks’ gestation who underwent emergent primary
a. White race cesarean delivery for suspected fetal distress in her
first pregnancy. Her uterine incision closure from
b. Single mother
the time of surgery is pictured. If she chooses a trial
c. Increased maternal age of labor for this pregnancy, what is her absolute risk
d. Short interdelivery interval of uterine rupture?
31–6. You are caring for a 26-year old G2P1 who 31–10. You see a patient at 34 weeks’ gestation. She had a
presents at 39 weeks’ gestation in active labor. She primary low-transverse cesarean delivery for failure
has a history of prior cesarean delivery for breech to progress 18 months ago. On ultrasound, the
presentation in her last pregnancy. This pregnancy lower uterine segment thickness measures 2.6 mm.
SECTION 8
has otherwise been uncomplicated. What is her How should you counsel the patient based on the
approximate chance of a successful trial of labor? sonographic findings?
a. 30% a. This finding has no prognostic significance.
b. 55% b. This finding is associated with a low risk of
c. 75% uterine rupture.
d. 95% c. This finding is associated with a high risk of
uterine rupture.
31–7. What is the chance of uterine rupture based on the d. This finding is associated with an intermediate
scar type shown in the photograph? risk of uterine rupture.
a. 0.2–0.9%
b. 1–7%
c. 2–6%
d. 2–9%
31–8. The majority of women delivering via cesarean a. Her risk of uterine rupture is negligible.
delivery prior to 26 weeks’ gestation require what
b. Her risk of uterine rupture is decreased because of
type of uterine incision?
the ultrasound findings.
a. Classical incision
c. Her risk of uterine rupture is unchanged because
b. Pfannenstiel incision of the ultrasound findings.
c. Low-vertical incision d. Her risk of uterine rupture is further increased
d. Low-transverse incision because of the ultrasound findings.
31–9. You are performing a primary low-transverse 31–12. Which risk factor confers the highest risk of uterine
cesarean delivery on a term patient for failure rupture in a subsequent pregnancy?
to progress. Which of the following operative a. Interdelivery interval <6 months
techniques may decrease her risk of uterine rupture
b. Prior upper segment uterine rupture
in a subsequent pregnancy?
c. Maternal body mass index >30 kg/m2
a. Locking stitches
d. Smallest myometrial thickness <2.0 mm
b. Double-layer closure
c. Placement of Floseal over the hysterotomy
d. None of the above
31–13. What is the approximate chance of successful vaginal 31–16. Which of the following statements is true regarding
birth after cesarean delivery in a patient with a body elective repeat cesarean deliveries based on the figure
mass index exceeding 40 kg/m2? that is shown?
CHAPTER 31
a. 50% 20 Any adverse outcome
b. 60% RDS or TTN
c. 70% Sepsis
d. 80% 15
Percent
who attempts a trial of labor with a 32-week preterm
gestation? 10
31–19. Which of the following has been demonstrated in 31–23. Uterine scar rupture can mimic which of the follow-
women with a labor epidural who have an increased ing obstetric emergencies?
risk of uterine rupture? a. Placental abruption
a. Adequate pain relief is never achieved
SECTION 8
b. Pulmonary embolus
b. More frequent epidural dosing is required c. Amniotic fluid embolus
c. Successful epidural placement is more difficult d. All of the above
d. None of the above
31–24. Which of the following findings is not helpful to
31–20. What percentage of successful vaginal births after diagnose a uterine rupture during labor?
cesarean delivery were completed via operative a. Fetal distress
assistance with either vacuum or forceps?
b. Uterine tenderness
a. 5%
c. A firmly contracted uterus
b. 15%
d. Decreased resting tone based on an intrauterine
c. 25% pressure catheter reading
d. 35%
31–25. As you are caring for a laboring patient at term who
31–21. Which of the following statements is true regarding is attempting a vaginal birth after having a previous
uterine scar exploration following successful vaginal cesarean delivery, you observe sudden and prolonged
birth after previous cesarean delivery? fetal bradycardia based on a fetal scalp electrode
a. Routine scar examination is recommended. monitor. You proceed with an emergent cesarean
delivery with the intraabdominal findings pictured.
b. The chance of identifying a defect is <0.1%.
The fetus has been extruded into the abdominal
c. Any identified defect requires immediate explor- cavity. What are the chances of fetal survival in this
atory laparotomy. situation?
d. The need for exploratory laparotomy is deter-
mined by the extent of the defect and the
presence of active bleeding.
Reproduced with permission from Cunningham FG, Leveno KJ, Bloom SL, et al (eds):
Obstetrical hemorrhage. In Williams Obstetrics, 25th ed. New York, McGraw-Hill, 2018,
Figure 41-13.
Reproduced with permission from Cunningham FG, Leveno KJ, Bloom SL, et al (eds): Prior a. 5–15%
cesarean delivery. In Williams Obstetrics, 25th ed. New York, McGraw-Hill, 2018, Figure 31-5.
b. 10–25%
a. Administer ephedrine c. 25–50%
b. Reposition the patient d. 50–75%
c. Attempt forceps placement
d. Proceed with emergent cesarean delivery
31–26. According to published studies, what is the critical 31–29. In a conservative approach to trial of labor follow-
time threshold from decision to delivery to avoid ing cesarean delivery, which of the following would
long-term neurological impairments in the infant? not be considered cautionary before pursuing a labor
CHAPTER 31
a. Less than 9 minutes induction in a patient with a history of cesarean
delivery?
b. Less than 18 minutes
a. High station
c. Less than 31 minutes
b. A Bishop score of 10
d. Less than 45 minutes
c. Unknown incision type
31–27. In a developed country, what are the risks of mater- d. Closed cervix at 41 weeks
nal and neonatal mortality, respectively, if a uterine
rupture does occur during labor? 31–30. When should discussion of the risks and benefits
a. 0.2% and 5% of a trial of labor following cesarean delivery ideally
begin?
b. 0.5% and 7%
a. Preconceptionally
c. 0.2% and 7%
b. At the first prenatal visit
d. 0.5% and 5%
c. Prior to the start of the second trimester
31–28. Which of the following statements is true based on d. After the patient presents to labor and delivery
the figure that is shown? with contractions
8
Wound/uterine infection 31–31. Which of the following led to the decrease in rates of
Placenta previa trial of labor after cesarean delivery after 1996?
7 Transfusion a. Reports of maternal mortality from uterine
Hysterectomy rupture
6 Placenta accreta b. Reports of perinatal mortality from uterine
Risk for complication (percent)
rupture
5 c. American College of Obstetricians and Gynecolo-
gists recommended that a physician be immediately
available
4
d. All of the above
3 31–32. Which of the following is associated with the highest
vaginal birth after cesarean delivery rate?
2 a. Prior vaginal delivery
b. Normal body mass index
1 c. Estimated fetal weight <4000 grams
d. Prior cesarean delivery for a nonrecurring
0 indication
First Second Third Fourth ≥ Fifth
(6201) (15,808) (6324) (1452) (347)
Number of repeat cesarean deliveries
(Number of women)
Reproduced with permission from Cunningham FG, Leveno KJ, Bloom SL, et al (eds):
Prior cesarean delivery. In Williams Obstetrics, 25th ed. New York, McGraw-Hill, 2018,
Figure 31-6.
THE NEWBORN
CHAPTER 32
32–1. After delivery, clearance of the amnionic fluid that 32–4. Which of the following is true regarding neonatal
fills the fetal lungs occurs via which of the following resuscitation?
mechanisms? a. Approximately 1% of newborns need extensive
a. Physical compression of the fetal thorax resuscitation after delivery.
b. Absorption of fluid into the neonatal pulmonary b. Newborns delivered at home face risk of death
circulation that is 5-fold that of newborns delivered in the
c. Absorption of fluid into the neonatal pulmonary hospital.
lymphatic system c. Approximately 30% of newborns require some
d. All of the above degree of active resuscitation to stimulate
breathing.
32–2. Which of the following postnatal changes promotes d. All of the above
closure of the ductus arteriosus in the newborn?
a. Fall in pulmonary arterial blood pressure 32–5. How can primary and secondary apnea in the
newborn be distinguished from each other?
b. Fall in cardiac output from reduced preload after
umbilical cord clamping a. Stimulation usually reverses primary apnea.
c. Increase in pulmonary vascular compression after b. Secondary apnea is accompanied by a fall in heart
lung aeration from neonatal inspiration rate and primary is not.
d. All of the above c. Secondary apnea is accompanied by loss of
neuromuscular tone and primary is not.
32–3. Which of the following is not a benefit of delayed d. All of the above
cord clamping, as pictured below?
32–6. A 26-year-old multigravida presents in spontaneous
labor at 39 weeks 3 days. She had spontaneous
rupture of membranes with clear amnionic fluid and
delivered spontaneously with a category I fetal heart
rate tracing. The neonate was vigorous and crying
immediately after delivery. What is the next best step
in the management of this newborn?
a. Bulb suctioning of oropharyngeal secretions
b. Cord clamping and transfer to the radiant warmer
c. Placement on maternal abdomen for drying and
evaluation of tone, heart rate, and respiratory effort
d. Hold infant in fully supported prone position
and deliver 3 gentle back thumps to aid in initial
secretion clearance.
32–8. The newborn in Question 32–6 was given positive 32–11. Which site in the image below depicts the correct
pressure ventilation. Nonetheless, the newborn’s location of fingers for chest compressions in the
heart rate at 80 seconds of life is 86 beats per neonate?
CHAPTER 32
minute. What is the next most appropriate step?
a. Chest compressions
b. Placement of an endotracheal tube
c. Placement of an umbilical vein catheter
d. Mask adjustment and airway repositioning
32–14. A term newborn has a heart rate of 128 beats per 32–18. A 27-year-old primigravida at 33 weeks’ gestation
minute, is pink except for her feet and hands, is has been on the antepartum unit for 6 days for
crying spontaneously and loudly, and is kicking preterm rupture of membranes. An image from
both legs. What is her Apgar score? her admission ultrasound is shown below. You
SECTION 9
32–19. A 23-year-old primigravida at 38 weeks’ gestation 32–21. Ms. Smith presents for her 6-week postpartum visit.
presents to labor and delivery after a witnessed seizure She is doing well, and is excited to show you her
at home. Her initial blood pressure is 130/80 mmHg, newborn daughter, who is doing well overall. She
CHAPTER 32
and pulse is 140 beats per minute. The fetal heart has noticed some eye drainage the last few days, and
rate tracing shows a heart rate of 100 beats per you observe the findings depicted below. Which of
minute, absent variability, and late decelerations. the following statements is true?
She is having frequent contractions, and exam shows
she is 3 cm dilated. She is taken back for emergent
cesarean delivery, and bloody fluid is noted upon
uterine entry. Placental findings are depicted below.
Umbilical artery blood gas pH is 6.9 and base deficit
is 14 mEq/L. Five-minute Apgar score is 2. Which
of the following statements is evidence-based?
32–23. Why is the infant below receiving an injection of 32–25. During her first trimester, Ms. Harris traveled to
vitamin K within 1 hour of birth? a country that is known to be an endemic area for
Zika virus. Subsequently her serological testing was
positive. Ultrasound surveillance of the fetus has
SECTION 9
32–26. Ms. Blake comes to see you on postpartum day 5 for 32–27. A 31-year-old primigravida undergoes a spontaneous
a blood pressure check. She had a spontaneous vagi- vaginal delivery at 40 weeks’ gestation, delivering
nal delivery at term complicated by preeclampsia and a female infant weighing 7 lb 9 oz. They are both
CHAPTER 32
chorioamnionitis. The neonate never had a fever and discharged home on postpartum day 2. She has been
has been feeding, voiding, and stooling well since home for 2 days and calls your office worried that
discharge. Ms. Blake shows you the umbilical cord the baby weighs 7 lb today. What is the most
stump, which is depicted below. What is the appro- appropriate response?
priate response to her concern?
Used with permission from Kelly Yanes. Used with permission from Dr. David Nelson.
a. Provide reassurance of the normal finding with a. If any further weight loss occurs, supplement
no new recommendations. breastfeeding with formula until her weight
stabilizes.
b. Recommend covering the stump with petroleum
jelly and a gauze pad to hasten cord separation. b. Recommend converting to a high calorie
commercial formula and schedule a weight check
c. Recommend she proceed to the emergency
with the pediatrician in 3 days.
room due to concern for necrosis and need for
resection. c. This amount of weight loss is within expectations.
d. Recommend she proceed to her pediatrician’s Observe wet diapers and continue breastfeeding
office due to concern for omphalitis with need for with expectation to regain birthweight by
antibiotics. 10 days.
d. None of the above
32–30. Ms. Lewis is pregnant with a male fetus. She asks 32–32. Which of the following is true regarding hospital
you about circumcision. Which of the following is stay after delivery?
not true? a. Early discharge is associated with increased
a. Newborn male circumcision rate is estimated to
SECTION 9
neonatal mortality.
be approximately 55% in the latest Centers for b. Early discharge is associated with increased
Disease Control report. neonatal readmission rates for dehydration and
b. Circumcision lowers the incidence of human jaundice.
papilloma virus related penile cancer in males and c. The Newborns’ and Mothers’ Health Protection
cervical cancer in their partners. Act of 1996 prohibits insurers from restricting
c. In their 2012 policy statement, the American hospital stays to less than 2 days for vaginal
Academy of Pediatrics Task Force on Circumcision delivery or 4 days for cesarean delivery.
concluded that health benefits of male circumcision d. All of the above
outweigh risks and recommended the procedure
for all newborns.
d. All of the above
CHAPTER 32
number answer cited Header cited
32–1 d p. 607 Transition to Air Breathing
32–2 a p. 607 Transition to Air Breathing
32–3 c p. 607 Transition to Air Breathing
Umbilical Cord Clamping
32–4 a p. 607 Newborn Resuscitation
32–5 a p. 608 Newborn Resuscitation
32–6 c p. 608 Resuscitation Protocol
32–7 d p. 609 Figure 32-2
32–8 d p. 610 Table 32-1
32–9 c p. 608 Alternative Airway
32–10 b p. 609 Chest Compressions
32–11 c p. 609 Chest Compressions
32–12 a p. 610 Epinephrine
32–13 b p. 610 Apgar Score
32–14 c p. 610 Apgar Score
32–15 d p. 611 Apgar Score
32–16 a p. 611 Fetal Acid–Base Physiology
32–17 b p. 612 Table 32-3
32–18 b p. 612 Figure 32-4
p. 613 Respiratory Acidemia
32–19 c p. 612 Clinical Significance of Acidemia
32–20 a p. 613 Eye Infection Prophylaxis
32–21 a p. 613 Eye Infection Prophylaxis
32–22 c p. 614 Hepatitis B Immunization
32–23 b p. 614 Vitamin K
32–24 d p. 614 Newborn Screening
32–25 c p. 614 Zika Virus
32–26 a p. 615 Care of Skin and Umbilical Cord
32–27 c p. 615 Feeding and Weight Loss
32–28 c p. 615 Feeding and Weight Loss
32–29 b p. 615 Stools and Urine
32–30 c p. 615 Male Circumcision
32–31 d p. 616 Male Circumcision
32–32 d p. 616 Rooming In and Hospital Discharge
CHAPTER 33
33–1. Which of the following is the most common reason 33–6. Which of the following forms of cerebral palsy can
for respiratory distress in term infants? result from acute peripartum ischemia?
a. Severe asphyxia a. Ataxia
b. Infection/sepsis b. Hemiplegia
c. Meconium aspiration c. Spastic diplegia
d. Elective cesarean delivery d. Spastic quadriplegia
33–2. What is the incidence of meconium-stained 33–7. Which of the following is not consistent with an
amnionic fluid in term laboring women? acute peripartum or intrapartum event leading to
a. 1–2% hypoxic ischemic encephalopathy?
b. 10–20% a. Umbilical artery pH <7
c. 30–50% b. Multisystem organ injury
d. 60–70% c. Apgar of >7 at 5 and 10 minutes
d. Sentinel hypoxic or ischemic event immediately
33–3. Which of the following is effective at reducing before or during delivery
meconium aspiration syndrome?
a. Amnioinfusion 33–8. Which of the following is the best imaging modality
for visualizing the neonatal brain in cases of suspected
b. Oropharyngeal suctioning
hypoxic-ischemic encephalopathy?
c. Cesarean delivery to avoid fetal heart tracing
a. Cranial ultrasound
abnormalities
b. Computed tomography
d. None of the above
c. Magnetic resonance imaging
33–4. Which of the following has been used for the d. All listed modalities are equally good
treatment of meconium aspiration syndrome?
a. Intubation 33–9. All except which of the following are considered
sentinel events when assessing for hypoxic-ischemic
b. Inhaled corticosteroids
encephalopathy?
c. Extracorporeal membrane oxygenation
a. Nuchal cord
d. All of the above
b. Uterine rupture
33–5. In order to be diagnosed with neonatal encephalopathy, c. Amnionic fluid embolus
a neonate must be born at a minimum of what d. Severe placental abruption
gestational age?
a. 24 weeks’ gestation 33–10. Which of the following is not a risk factor for neonatal
acidosis?
b. 28 weeks’ gestation
a. Chorioamnionitis
c. 32 weeks’ gestation
b. Regional anesthesia
d. 35 weeks’ gestation
c. Advanced maternal age
d. Emergency cesarean delivery
33–11. What is the prevalence of cerebral palsy in the 33–16. Which of the following statements about
United States? neuroimaging studies for neonatal encephalopathy
a. 1/1000 children and cerebral palsy is true?
CHAPTER 33
b. 2/1000 children a. They can precisely time injuries.
c. 10/1000 children b. Findings are not dependent on gestational age.
d. 50/1000 children c. Magnetic resonance imaging findings correlate
with severity of disability.
33–12. What is the single most important risk factor for d. Sonographic studies and computed tomography
cerebral palsy? scans are generally normal on the first day of life.
a. Hydramnios
33–17. Which of the following is a major predictor of
b. Preterm birth seizure disorders?
c. Maternal obesity a. Neonatal seizures
d. Chorioamnionitis b. Fetal malformations
33–13. What fetal heart abnormality predicts cerebral palsy c. Family history of seizure disorder
and should thus prompt emergent intervention if d. All of the above
present?
a. Tachysystole 33–18. Which of the following is not seen in neonatal
abstinence syndrome?
b. Variable decelerations
a. Seizures
c. Minimal beat-to-beat variability
b. Irritability
d. No specific fetal heart rate pattern predicts
cerebral palsy c. Hypotonia
d. Poor suck reflex
33–14. You are seeing a patient and her husband after
the delivery of their first child. The patient had 33–19. Which of the following is a known complication of
preeclampsia with severe features and a placental delayed cord clamping?
abruption for which you performed an emergency a. Plethora
cesarean delivery. The couple is very concerned b. Petechiae
about their infant. You want them to be counseled
thoroughly by the neonatologists, but they are c. Neonatal anemia
anxious for any information you can give them on d. Hyperbilirubinemia
their baby’s outcome. You know that the 10-minute
Apgar score was 2. Based on that, what is the risk of 33–20. When do serum bilirubin levels peak in newborns?
cerebral palsy? a. Day of life 1
a. 1% b. Day of life 2
b. 5% c. Day of life 3–4
c. 10% d. Day of life 7–10
d. 25%
33–21. The infant pictured below is being treated for neona- 33–25. Which of the following carries the highest incidence
tal hyperbilirubinemia. What is first-line treatment? of major birth trauma?
a. Failed forceps delivery
SECTION 9
33–22. Which of the following is not a vitamin 33–28. Present at birth and gone within hours or days, the
K–dependent clotting factor? schematic below depicts which of the following?
a. Factor V
b. Factor VI
c. Factor VII
d. Factor XIII
33–29. With mortality rates of 12–18% from blood loss, the 33–31. A 17-year-old G1P1 presented at term in labor. She
schematic below depicts which of the following? progressed to complete dilation. After pushing for
3 hours, she was taken for a cesarean delivery for
CHAPTER 33
failure to descend. The head was wedged deep in the
pelvis. An assistant had to provide a vaginal hand to
dislodge the head. After birth, a radiograph of the
neonate’s head was performed and is shown below.
What is the diagnosis?
Reproduced with permission from Cunningham FG, Leveno KJ, Bloom SL, et al (eds):
Diseases and injuries of the term newborn. In William Obstetrics, 25th ed. New York,
McGraw-Hill, 2018, Figure 33-2.
a. Preparietal bleed
b. Cephalohematoma
c. Caput succedaneum
d. Subgaleal hemorrhage
Reproduced with permission from Cunningham FG, Leveno KJ, Bloom SL, et al (eds):
Diseases and injuries of the term newborn. In William Obstetrics, 25th ed. New York,
McGraw-Hill, 2018, Figure 33-3.
a. Linear fracture
b. Craniosynostosis
Reproduced with permission from Cunningham FG, Leveno KJ, Bloom SL, et al (eds):
Diseases and injuries of the term newborn. In William Obstetrics, 25th ed. New York,
c. Occipital osteodiastasis
McGraw-Hill, 2018, Figure 33-2. d. Depressed skull fracture
a. Preparietal bleed 33–32. A 24-year-old G1P1 gave birth to a term infant. The
b. Cephalohematoma infant weighed 9 pounds at birth. The pediatrician
noted that the newborn’s left arm was held straight
c. Caput succedaneum
and internally rotated, the elbow was extended, and
d. Subgaleal hemorrhage the wrist and fingers were flexed. The fingers did
move freely. What nerve root is affected?
a. C4
b. C5-6
c. C8-T1
d. None of the above
33–33. A 22-year-old G1P1 presented at term in active 33–34. Which of the following neonatal long-bone fractures
labor. She had an uncomplicated vaginal delivery. is the most common?
A picture of her newborn is provided below. What a. Femoral
nerve was injured?
SECTION 9
b. Humeral
c. Clavicular
d. Mandibular
a. Facial
b. Optic
c. Vagus
d. Trigeminal
CHAPTER 33
number answer cited Header cited
33–1 b p. 619 Respiratory Distress Syndrome
33–2 b p. 620 Meconium Aspiration Syndrome
33–3 d p. 620 Prevention
33–4 d p. 620 Treatment
33–5 d p. 621 Neonatal Encephalopathy
33–6 d p. 621 Neonatal Encephalopathy
33–7 c p. 621 Table 33-1
33–8 c p. 622 Criteria for Hypoxic-Ischemic Encephalopathy
33–9 a p. 622 Criteria for Hypoxic-Ischemic Encephalopathy
33–10 b p. 622 Criteria for Hypoxic-Ischemic Encephalopathy
33–11 b p. 622 Cerebral Palsy
33–12 b p. 623 Incidence and Epidemiological Correlates
33–13 d p. 623 Intrapartum Fetal Heart Rate Monitoring
33–14 c p. 624 Apgar Scores
33–15 c p. 624 Umbilical Cord Blood Gas Studies
33–16 d p. 624 Neuroimaging in Neonatal Period
33–17 d p. 625 Intellectual Disability and Seizure Disorders
33–18 c p. 625 Neonatal Abstinence Syndrome
33–19 d p. 625 Anemia
33–20 c p. 626 Hyperbilirubinemia
33–21 b p. 626 Prevention and Treatment
33–22 b p. 626 Hemorrhagic Disease of the Newborn
33–23 d p. 626 Hemorrhagic Disease of the Newborn
33–24 b p. 627 Immune Thrombocytopenia
33–25 b p. 627 Table 33-4
33–26 b p. 628 Intracranial Hemorrhage
33–27 d p. 628 Table 33-5
33–28 c p. 629 Figure 33-2
33–29 d p. 629 Figure 33-2
33–30 b p. 629 Figure 33-2
33–31 d p. 629 Skull Fractures
33–32 b p. 630 Brachial Plexopathy
33–33 a p. 630 Figure 33-4
33–34 c p. 630 Fractures
CHAPTER 34
34–1. Compared to term infants, neonates born prematurely 34–6. What is the etiology of the diffuse reticulogranular
have higher associated rates of which of the following? infiltrate seen on the chest radiograph below?
a. Sleep apnea
b. Developmental delay
c. Congenital malformations
d. All of the above
34–7. A 32-year-old primigravida delivers a neonate at 34–10. A 37-year-old multigravida presents for a 6-week
31 weeks’ gestation due to preeclampsia with postpartum visit after delivering her son prematurely
severe features. She received a course of antenatal at 28 weeks’ gestation. She informs you that he
CHAPTER 34
corticosteroids prior to delivery, but in the delivery developed severe respiratory distress syndrome and
room the neonate receives the intervention pictured was on a mechanical ventilator for many weeks.
below. The neonate likely displayed all except which She is worried he will develop bronchopulmonary
of the following clinical signs? dysplasia (BPD). Which of the following medications
have not been shown to prevent BPD?
a. Caffeine
b. Vitamin A
c. Glucocorticoids
d. Inhaled nitric oxide
34–15. At what gestational age does the concentration of 34–19. A 29-year-old primigravida underwent a cesarean
lecithin relative to sphingomyelin rise? delivery of a female infant at 32 weeks’ gestation
a. 23 weeks secondary to preeclampsia with severe features and
fetal-growth restriction. The infant weighed 1498
SECTION 9
b. 29 weeks
grams at birth. Which of the characteristics of this
c. 30 weeks infant, pictured below, is not a risk factor for the
d. 34 weeks development of necrotizing enterocolitis?
a. Intestinal immaturity
b. Exposure to enteral feeds
c. Highly immunoreactive intestinal mucosa
d. All of the above
34–21. How does hyperoxemia lead to retinopathy of 34–25. Which of the following is the preferred initial
prematurity? modality for identifying brain abnormalities in the
a. Causes centripetal vascularization of retina newborn?
CHAPTER 34
b. Causes hemorrhage from retinal vessels and a. Sonography
subsequent aberrant neovascularization b. Computed tomography
c. Causes severe retinal vasoconstriction, c. Magnetic resonance imaging
vessel obliteration, and subsequent aberrant d. Positron emission tomography
neovascularization
d. Causes dilation of the retinal vessels, increase 34–26. Which of the following contributes to the increased
in angiogenic factors, and subsequent aberrant risk of intraventricular hemorrhage in preterm
neovascularization infants?
a. Venous stasis and congestion
34–22. The infant pictured below was born at 34 weeks’
b. Impaired vascular autoregulation
gestation by cesarean delivery due to preeclampsia
with severe features. The infant’s birthweight c. Poor vessel support by the subependymal
was 1928 grams. Even at this gestational age, low germinal matrix
birthweight is a risk factor for which of the following d. All of the above
complications of prematurity?
34–27. A 34-year-old multigravida presents for her post
partum visit and informs you her infant, born at
25 weeks’ gestation, has a bleed in its brain. She
reports she was told the bleeding has extended into
the tissue. What grade intraventricular hemorrhage
does her infant have?
a. Grade I
b. Grade II
c. Grade III
d. Grade IV
34–30. All except which of the following are risk factors for 34–32. Which of the following statements about periven-
development of cerebral palsy? tricular leukomalacia are true?
a. Ischemia a. It is associated with cerebral palsy
SECTION 9
CHAPTER 34
number answer cited Header cited
34–1 c p. 636 Introduction
34–2 b p. 636 Introduction
34–3 d p. 636 Respiratory Distress Syndrome
34–4 c p. 637 Etiopathogenesis
34–5 c p. 637 Etiopathogenesis
34–6 c p. 637 Clinical Course
34–7 c p. 637 Treatment
34–8 d p. 637 Treatment
34–9 d p. 637 Clinical Course
34–10 d p. 637 Treatment
34–11 b p. 638 Surfactant Prophylaxis and Rescue
34–12 d p. 638 Surfactant Prophylaxis and Rescue
34–13 b p. 638 Prevention
34–14 a p. 638 Prevention
34–15 d p. 638 Prevention
34–16 d p. 638 Prevention
34–17 c p. 638 Prevention
34–18 b p. 638 Prevention
34–19 a p. 639 Necrotizing Enterocolitis
34–20 d p. 639 Necrotizing Enterocolitis
34–21 c p. 639 Retinopathy of Prematurity
34–22 a p. 639 Necrotizing Enterocolitis
34–23 b p. 639 Brain Disorders
34–24 a p. 639 Intracranial Hemorrhage
34–25 a p. 639 Brain Disorders
34–26 d p. 639 Periventricular-Intraventricular Hemorrhage
34–27 d p. 640 Periventricular-Intraventricular Hemorrhage
34–28 d p. 640 Periventricular-Intraventricular Hemorrhage
34–29 d p. 641 Periventricular-Intraventricular Hemorrhage
34–30 c p. 641 Risks
34–31 d p. 641 Cerebral Palsy
34–32 d p. 640 Periventricular Leukomalacia
34–33 c p. 641 Risks
CHAPTER 35
Stillbirth
35–1. When a standardized evaluation including autopsy, 35–6. A 28-year-old primigravida with uncomplicated
placental pathology, and testing of maternal and prenatal care presents to the labor unit at 38 weeks’
fetal tissues such as karyotype is conducted, in what gestation with contractions and decreased fetal
percent of stillbirths can a probable or possible cause movement. A diagnosis of fetal demise is made.
be identified? Which of the following is true regarding establishing
a. 34% a cause of stillbirth?
b. 58% a. May aid maternal coping
c. 76% b. Allows accurate counseling regarding recurrence
risk
d. 90%
c. May prompt therapy or intervention to prevent a
35–2. Which of the following requirements must be met similar outcome in a future pregnancy
for reporting a fetal death in all U.S. states? d. All of the above
a. Weight >350 grams
b. Weight >500 grams 35–7. The patient in Question 35–6 declines autopsy but
remains undecided regarding chromosomal analysis.
c. Gestational age >20 weeks Gross examination of the fetus reveals no obvious
d. None of the above—the requirements vary by abnormalities or dysmorphology. How should the
state patient be counseled?
a. Chromosomal analysis is not indicated.
35–3. What was the most common cause of stillbirth in
the Stillbirth Collaborative Research Network Study? b. The risk of a chromosomal abnormality is up to
5%.
a. Undetermined
c. The risk of a chromosomal abnormality is up to
b. Fetal malformations 14%.
c. Placental abnormalities d. The risk of a chromosomal abnormality is up to
d. Obstetrical complications 25%.
35–4. Which of the following is considered a risk factor for 35–8. Recently, chromosomal microarray became the
stillbirth? recommended method for chromosomal analysis in
a. Obesity the setting of stillbirth. Which of the below state-
ments regarding the benefit of chromosomal micro-
b. Nulliparity
array compared to traditional karyotype is correct?
c. Advanced maternal age
a. Chromosomal microarray provides quicker results
d. All of the above
b. Chromosomal microarray results are more
accurate
35–5. Which of the following maternal risk factors carries
the highest risk of stillbirth? c. Chromosomal microarray results are more easily
interpreted
a. Prior stillbirth
b. Chronic hypertension d. Chromosomal microarray does not require
dividing cells
c. Cholestasis of pregnancy
d. Systemic lupus erythematosus
35–9. A 25-year-old primigravida presents at 37 weeks’ 35–12. How often do autopsy results change the recurrence
gestation with decreased fetal movement and is risk estimates and parental counseling following
diagnosed with a fetal demise. On ultrasound stillbirth?
CHAPTER 35
amnionic fluid appears normal. She undergoes a. 5–10% of cases
induction of labor without complications and
b. 10–20% of cases
delivers a stillborn infant that appears normal
on examination. Examination of the placenta c. 25–50% of cases
demonstrates no obvious abnormalities. Which of d. 50–75% of cases
the maternal serum tests below is not indicated?
a. Glucose level 35–13. A 35-year-old primigravida presents with fetal
demise at 30 weeks’ gestation. Which of the
b. Kleihauer-Betke
following may be associated with poor coping and
c. Type and screen greater risk of postpartum depression?
d. Factor V Leiden testing a. Lack of keepsake items or photos
b. Having her infant taken away due to need for
35–10. When a patient declines autopsy following stillbirth,
testing
which of the following tests may be useful, in
addition to a fetogram such as the one shown below? c. Prolonged interval between diagnosis and induction
d. All of the above
a. Photography
b. Bacterial cultures
c. Magnetic resonance imaging
d. All of the above
a. Diabetes mellitus
b. Placental insufficiency
c. Fetal growth restriction
d. None of the above
35–15. Which of the following recommendations could be 35–20. A 41-year-old G6P5 presents for fetal sonographic
made to the patient in Question 35–14 to decrease evaluation at 19 weeks’ gestation. The following
her future risk of recurrent stillbirth? image is obtained, which demonstrates an absent
a. Maternal karyotype calvarium (an arrow indicates the chin and asterisks
SECTION 9
35–21. The patient in Question 35–5 presents at 35 weeks’ 35–22. A 40-year-old G6P5 presents at 36 weeks’ gestation
gestation for a prenatal care visit and is found to with complaints of contractions and vaginal bleeding.
have a stillbirth. Induction is undertaken, and she Her prenatal care was routine other than admin-
CHAPTER 35
delivers the infant pictured below. Based on the istration of anti-D immune globulin at 28 weeks’
Stillbirth Collaborative Research Writing Group’s gestation. On admission her blood pressure is
categories, how would this stillbirth be classified 166/98 mmHg and proteinuria is found on urinalysis.
with regard to the underlying cause? Her cervix is 8 cm dilated and there is active vaginal
bleeding. No fetal heart tones can be found, and a
stillbirth is confirmed by sonographic examination.
At delivery the placenta is noted to have the following
appearance. What is the most common associated
risk factor for this condition?
a. Possible
b. Definite
c. Probable
d. Unknown a. Hypertension
b. Hypothyroidism
c. Grand multiparity
d. Advanced maternal age
35–24. For the patient in Question 35–21, which additional 35–27. At what gestational age should antenatal testing
maternal blood test would be most useful in this begin in women with a history of prior stillbirth?
clinical situation? a. 28 weeks
a. Kleihauer-Betke test
SECTION 9
b. 32 weeks
b. Thrombophilia testing c. 34 weeks
c. Lupus anticoagulant testing d. 38 weeks
d. Serum glucose measurement
35–28. A 34-year-old G2P1 presents for prenatal care at 8
35–25. Which specimen is most desirable for chromosomal weeks’ gestation. She reveals that her first pregnancy
microarray analysis of the stillborn infant? ended in a stillbirth at 36 weeks’ gestation. At what
a. Amnionic fluid gestational age do you recommend she undergo
delivery during this pregnancy?
b. Fetal cord blood
a. 34 weeks
c. Placental tissue sample
b. 36 weeks
d. Umbilical cord segment
c. 38 weeks
35–26. Almost half of all fetal deaths are associated with d. 39 weeks
what pregnancy complication?
a. Preeclampsia
b. Oligohydramnios
c. Fetal malformations
d. Fetal growth restriction
CHAPTER 35
number answer cited Header cited
35–1 c p. 645 Causes of Fetal Death
35–2 d p. 645 Definition of Fetal Mortality
35–3 d p. 646 Causes of Fetal Death
35–4 d p. 646 Risk Factors
35–5 d p. 647 Risk Factors
35–6 d p. 647 Evaluation of the Stillborn Fetus
35–7 b p. 647 Evaluation of the Stillborn Fetus
35–8 d p. 647 Evaluation of the Stillborn Fetus
35–9 d p. 648 Evaluation of the Stillborn Fetus
35–10 d p. 648 Evaluation of the Stillborn Fetus
35–11 c p. 648 Evaluation of the Stillborn Fetus
35–12 c p. 648 Evaluation of the Stillborn Fetus
35–13 d p. 648 Psychological Aspects
35–14 a p. 648 Prior Stillbirth
35–15 c p. 648 Prior Stillbirth
35–16 d p. 648 Prior Stillbirth
35–17 b p. 649 Prior Stillbirth
35–18 b p. 645 Figure 35-2
35–19 a p. 645 Definition of Fetal Mortality
35–20 a p. 646 Risk Factors
35–21 c p. 646 Causes of Fetal Death
35–22 a p. 646 Causes of Fetal Death
35–23 d p. 646 Evaluation of the Stillborn Fetus
35–24 a p. 646 Evaluation of the Stillborn Fetus
35–25 b p. 647 Evaluation of the Stillborn Fetus
35–26 d p. 648 Prior Stillbirth
35–27 b p. 649 Prior Stillbirth
35–28 d p. 648 Prior stillbirth
THE PUERPERIUM
CHAPTER 36
The Puerperium
36–1. What duration of time is encompassed in the 36–6. A 26-year-old G3P3 is postpartum day 1 following
puerperium? an uncomplicated vaginal delivery. She reports sharp,
a. 2–4 weeks intermittent lower abdominal pain, which is more
severe than in her prior deliveries. Her heart rate is
b. 4–6 weeks
84 beats per minute, blood pressure 110/60 mmHg,
c. 6–8 weeks and her temperature 99.3 °F. Her abdomen is soft
d. 10–12 weeks on exam and there is no uterine tenderness. A scant
amount of lochia is appreciated on bimanual exam.
36–2. What are myrtiform caruncles? What is the most likely diagnosis?
a. Vaginal rugae a. Afterpains
b. Scarred tags of hymenal tissue b. Endometritis
c. A sexually transmitted infection c. Bladder flap hematoma
d. Microscopic tears in the vaginal epithelium d. Septic pelvic thrombophlebitis
36–3. At what point in the puerperium does the 36–7. A 20-year-old G1P1 presents to the emergency room
endocervical canal reform? 10 days after an uncomplicated vaginal delivery
a. 1 week postpartum complaining of fever, nausea/vomiting, abdominal
pain, and increased vaginal discharge. On arrival, she
b. 2 weeks postpartum is febrile to 38.6 °C and she has fundal tenderness on
c. 3 weeks postpartum exam. Which of the following tests would be least
d. 4 weeks postpartum helpful for further evaluation?
a. A complete blood count
36–4. What percentage of women experience regression of b. A basic metabolic profile
high-grade dysplasia following delivery?
c. A transvaginal ultrasound
a. 10%
d. A Gram stain of her vaginal discharge
b. 33%
c. 35% 36–8. A 40-year-old G2P2 presents to your office 2 weeks
d. 50% after her scheduled cesarean delivery for follow-up.
She complains of intermittent episodes of heavy
36–5. How long does complete uterine involution take vaginal bleeding, occurring as recently as the day
following delivery? prior. Her temperature is 99.6°F, heart rate 98 beats
a. 1 week per minute, and blood pressure 120/80 mmHg. On
exam, her uterus is noted to be enlarged to 20 weeks
b. 2 weeks in size and boggy. Her incision appears well approxi-
c. 3 weeks mated without evidence of infection. The most
d. 4 weeks appropriate management includes which of the
following?
a. A complete blood count
b. Methergine administration
c. Empiric antibiotic treatment
d. All of the above
36–9. Secondary postpartum hemorrhage is defined as 36–14. Your patient is postoperative day 2 following a primary
uterine hemorrhage occurring during what time- cesarean delivery for failure to progress. She is frustrated
frame after delivery? because she is only getting small drops of thick yellow
CHAPTER 36
a. 24 hours to 6 weeks liquid from her breasts. How long would you tell her
to expect colostrum production before beginning her
b. 48 hours to 6 weeks
conversion to a more mature milk?
c. 24 hours to 12 weeks
a. 2 to 5 days
d. 48 hours to 12 weeks
b. 2 to 10 days
36–10. What are common features of the urinary bladder in c. 5 to 14 days
the postpartum period? d. 5 to 21 days
a. Increased capacity
b. Incomplete emptying 36–15. What vitamin is virtually absent in human breast
milk?
c. Insensitivity to intravesical pressures
a. Vitamin C
d. All of the above
b. Vitamin A
36–11. A 24-year-old G1P1 complains of abdominal pain c. Vitamin K
and subjective fever 1 day following an uncomplicated d. All vitamins are equally represented in breast milk
vaginal delivery. Which of the following physical
exam signs or laboratory values would be most 36–16. Which hormone is most responsible for milk
helpful to make a diagnosis of endometritis in this expression during lactation?
postpartum patient? a. Prolactin
a. Fundal tenderness b. Oxytocin
b. An absolute neutrophilia c. Dopamine
c. White blood cell count of 25,000/μL d. Progesterone
d. All of the above
36–17. How would you advise a patient who is exclusively
36–12. After delivery, how long do cardiovascular parameters breastfeeding 4 weeks after delivery and develops
including cardiac output, heart rate, and blood nipple fissures?
pressure take to return to nonpregnant levels? a. Wash the area with mild soap and water daily.
a. 48 hours b. Apply a steroid cream and use a nipple shield
b. 72 hours temporarily.
c. 7 days c. Continue exclusive breastfeeding without
d. 10 days interruption.
d. Do not allow infant to feed on the affected side
36–13. You counsel your puerperal patient that most women and empty the breast regularly with a pump.
first approach their prepregnancy weight by which
time interval following delivery? 36–18. All except which of the following conditions are
a. 3 months contraindications to breastfeeding?
b. 6 months a. Infant with galactosemia
c. 9 months b. Maternal hepatitis B infection
d. 12 months c. Human immunodeficiency virus infection
d. Maternal active and untreated tuberculosis
36–19. A 21-year-old G1P1 presents to your office 4 days fol- 36–20. Your patient presents 5 days postpartum with
lowing an uncomplicated vaginal delivery complaining an axillary mass. She noted it during pregnancy
of breast pain and difficulty with breastfeeding. Her although reports it was much smaller. Yesterday, she
temperature is 37.8oC, heart rate 102 beats per
SECTION 10
36–22. Your patient had a 4-hour second stage of labor and 36–25. Which of the following is true regarding the postpar-
vaginal delivery without laceration. As her epidural tum blues?
analgesia subsides, she complains of perineal pain. Her a. May be affected by body image concerns
CHAPTER 36
temperature is a 37.0oC, pulse 84 beats per minute,
b. Usually lasts for no more than 10 days after delivery
and blood pressure 120/68 mmHg. Her first void
yielded 300 mL of urine. Management of the patient c. Effective treatment involves recognition and
should primarily include which of the following? reassurance
a. Perineal cool pack d. All of the above
b. Surgical evacuation 36–26. A 30-year-old G1P1 complains of difficulty with
c. Diagnostic needle aspiration ambulation on postpartum day 1 following a labor
d. Broad-spectrum intravenous antibiotic therapy and delivery course, which lasted 24 hours and
included 3 hours of pushing in stirrups. You perform
36–23. A 19-year-old G1P1 complains of worsening vul- an exam and note bilateral foot drop on exam.
var pain in the recovery room 6 hours following a Injury to what nerve is the most likely cause of the
vaginal delivery. The delivery was complicated by a patient’s ambulation difficulty?
prolonged second stage and a third degree laceration. a. Femoral nerve
On further evaluation, her temperature is 37.2oC, b. Ilioinguinal nerve
heart rate 130 beats per minute, blood pressure
86/52 mmHg, and respiratory rate 28 breaths per c. Common peroneal nerve
minute. She appears pale and mildly diaphoretic. d. Lateral femoral cutaneous nerve
Examination of the perineum reveals the findings
shown in the photo below. What is the next best 36–27. Which of the following is true regarding the
course of action? condition shown in the pelvic radiograph?
Reproduced with permission from Cunningham FG, Leveno KJ, Bloom SL, et al (eds): The
a. Urgent surgical evacuation Puerperium. In Williams Obstetrics, 25th ed. New York, McGraw-Hill, 2018, Figure 36-7.
36–29. Your patient presents for her 6-week follow-up 36–32. Approximately what percentage of postpartum
visit after having a primary cesarean delivery for women require readmission to the hospital within
breech presentation. She is trying to breastfeed 8 weeks of delivery?
SECTION 10
CHAPTER 36
number answer cited Header cited
36–1 b p. 652 Introduction
36–2 b p. 652 Birth Canal
36–3 a p. 653 Uterus
36–4 d p. 653 Uterus
36–5 d p. 653 Uterus
36–6 a p. 654 Afterpains
36–7 d p. 654 Lochia
36–8 d p. 654 Subinvolution
36–9 c p. 654 Late Postpartum Hemorrhage
36–10 d p. 655 Urinary Tract
36–11 a p. 655 Hematological and Coagulation Changes
36–12 d p. 655 Pregnancy Induced Hypervolemia
36–13 b p. 656 Postpartum Diuresis
36–14 c p. 656 Breast Anatomy and Secretory Products
36–15 c p. 656 Breast Anatomy and Secretory Products
36–16 b p. 656 Endocrinology of Lactation
36–17 d p. 658 Care of Breasts
36–18 b p. 658 Contraindications to Breastfeeding
36–19 d p. 659 Breast Engorgement
36–20 b p. 659 Other Issues with Lactation
36–21 b p. 660 Hospital Care
36–22 a p. 660 Perineal Care
36–23 a p. 660 Perineal Care
36–24 b p. 660 Bladder Function
36–25 d p. 661 Pain, Mood and Cognition
36–26 c p. 661 Neuromusculoskeletal Problems
36–27 c p. 661 Musculoskeletal Injuries
36–28 d p. 662 Contraception
36–29 b p. 663 Contraception
36–30 b p. 663 Coitus
36–31 a p. 663 Coitus
36–32 b p. 663 Late Maternal Morbidity
36–33 a p. 663 Follow-up Care
CHAPTER 37
Puerperal Complications
37–1. What percentage of non-breastfeeding women 37–6. What organism has been implicated in late-onset,
develop fever from breast engorgement postpartum? indolent metritis?
a. 15% a. Proteus
b. 25% b. Klebsiella
c. 35% c. Chlamydia
d. 55% d. Peptostreptococcus
37–2. What is the most common etiology of persistent 37–7. What is the most important criterion for the
fevers after childbirth? diagnosis of postpartum metritis?
a. Atelectasis a. Fever
b. Pyelonephritis b. Leukocytosis
c. Breast engorgement c. Foul-smelling lochia
d. Genital tract infections d. Parametrial tenderness
37–3. What physiological process in the postpartum period 37–8. A 20-year-old G1P1 underwent a cesarean delivery
causes urinary tract infections to be uncommon? for failure to progress. Just prior to surgery, the
a. Diuresis patient was diagnosed with metritis and started on
broad-spectrum antibiotics, which were continued
b. Passage of lochia postpartum. The patient continues to have fever on
c. Uterine involution postoperative day 5. She does not appear septic. She
d. Immunosuppression is frustrated that she can’t go home. Which of the
following is the least likely diagnosis?
37–4. Which of the following is the single most significant a. Infected hematoma
risk factor for development of an uterine infection? b. Parametrial phlegmon
a. Route of delivery c. Septic pelvic thrombophlebitis
b. Use of internal monitors d. Antimicrobial-resistant bacteria
c. Artificial rupture of membranes
d. Group B streptococcus colonization 37–9. What is the difference in treatment for metritis in
cases of vaginal delivery versus cesarean section?
37–5. A 35-year-old multigravida at 39 weeks’ gestation a. Dosing of antibiotics
undergoes an induction for elevated blood pressures b. Coverage of anaerobes
at term. After 24 hours, placement of internal
monitors, and 10 cervical exams, the patient c. Duration of treatment
undergoes a cesarean delivery for failure to progress. d. Route of administration of antibiotics
Which of the following is not a risk factor for her
developing an uterine infection? 37–10. Which of the following regimens is the gold standard
a. Multiparity for treatment of a pelvic infection following a cesarean
delivery?
b. Prolonged labor
a. Meropenem
c. Cesarean delivery
b. Vancomycin
d. Placement of internal monitors
c. Clindamycin and aztreonam
d. Clindamycin and gentamicin
37–11. A 22-year-old primigravida at term is diagnosed with 37–15. A 30-year-old G2P2 presents on postoperative day
failure to progress, and the plan is made for cesarean 6 to the emergency room complaining of drainage
delivery. The patient has no medical problems other from her cesarean incision. The cesarean delivery was
CHAPTER 37
than a body mass index of 50 kg/m2. She also denies for failure to progress after a long induction. The
any allergies. The anesthesiologist asks if you would patient is currently afebrile. Her body mass index
like routine perioperative antimicrobial prophylaxis. is 47 kg/m2. The patient reports that she sat down
Which antibiotic do you ask for to further reduce yesterday and felt a pop. Shortly thereafter, she
her chance of a postoperative infection? noticed pink/light brown drainage coming from
a. 3 grams instead of 2 grams of cefazolin her incision. She endorses chills and skin irritation
around the incision. Lochia has been normal. On
b. Vancomycin instead of the routine cefazolin
your exam, her skin is erythematous near the incision.
c. Routine 2 grams of cefazolin prior to skin There is serosanguinous drainage from the wound.
incision Although the patient has good pain tolerance, why
d. Ampicillin, gentamicin, and clindamycin for do you elect to take her to the operating room to
24 hours after the surgery evaluate the incision?
a. You plan to proceed with hysterectomy.
37–12. Which of the following has not been shown to lower
b. You want to open the wound, debride necrotic
the risk for infection after cesarean delivery?
tissue, and then close it back up using en bloc
a. Spontaneous separation of the placenta closure.
b. Chlorhexidine-alcohol skin preparation c. You want to place a negative-pressure wound
c. Single-dose antibiotics prior to skin incision therapy system and that can only be done in the
d. Surgeons changing gloves after delivery of the operating room.
placenta d. You are concerned that the fascia may not be
intact and if so, the fascia needs to be closed in
37–13. In more than 90% of women, metritis responds to the operating room.
treatment with antibiotics within what period of
time? 37–16. Which of the following statements about necrotizing
a. 12–24 hours fasciitis is false?
b. 24–36 hours a. It is common with low mortality rates.
c. 48–72 hours b. Three risk factors are diabetes, obesity, and
hypertension.
d. 72–96 hours
c. Surgical debridement of infected tissue should
37–14. Which of the following is an evidence-based statement leave wide margins of healthy bleeding tissue.
about the use of vacuum-assisted wound closure d. Early diagnosis, surgical debridement, anti
devices in obstetrics? microbials, and intensive care are paramount to
a. It prevents wound infection. successful treatment.
b. It is superior to standard dressings.
37–17. Which of the following statements about an ovarian
c. Provider time is decreased substantially. abscess in the puerperium is true?
d. It is significantly more cost effective than stan- a. Rupture is rare
dard dressings.
b. Usually affects both ovaries
c. Women present 4–6 weeks after delivery
d. It is thought to be caused by bacterial invasion of
the ovary through a rent in the capsule.
37–18. Which of the following is frequently the first sign/ 37–21. What is the overall incidence of septic pelvic
symptom of peritonitis in a postpartum woman? thrombophlebitis?
a. Diarrhea a. 1/100
SECTION 10
37–19. Which of the following statements about the 37–22. A 19-year-old primigravida undergoes a cesarean
phlegmon illustrated in the figure below is true? delivery for failure to progress. Her course is com-
plicated by chorioamnionitis for which she receives
broad-spectrum antibiotics. On postoperative day 5,
the patient is still having fevers. She feels well and is
becoming annoyed that she can not go home. Her
incision is healing nicely with no erythema or drain-
age. On computed tomography imaging, the patient
is noted to have a clot that extends to the ovarian
vein. An image is provided below. What is the next
step in your management?
Reproduced with permission from Cunningham FG, Leveno KJ, Bloom SL, et al (eds):
Puerperal complications. In Williams Obstetrics, 25th ed. New York, McGraw-Hill, 2018, a. Therapeutic heparin
Figure 37-3.
b. Continuation of antibiotic therapy
a. This is usually a bilateral process. c. Consult interventional radiology about removing
the clot and/or placing a filter.
b. Rarely is this limited to the parametrium.
d. Stop all medications and discharge home, so she
c. Typically fever resolves in 5–7 days with broad- can walk, reducing further clot risk.
spectrum antibiotics.
d. The most common route of extension is posteri-
orly into the rectovaginal septum.
37–23. Which of the following is a risk factor for episiotomy 37–27. Which of the following is the best treatment for
dehiscence? toxic shock syndrome?
a. Smoking a. Supportive care
CHAPTER 37
b. Infection b. Supportive care and antibiotics
c. Genital warts c. Supportive care, antibiotics, and wound
d. All of the above debridement if necessary
d. There is no necessary treatment as it will resolve
37–24. A 22-year-old G1P1 presents 6 days after a on its own over time
vaginal delivery. Her course was complicated by
chorioamnionitis and a second-degree perineal 37–28. What is the incidence of mastitis?
laceration. The patient is complaining of pain and a. 1%
drainage from her vagina. On exam, her laceration
repair is open and draining purulent material. b. 3%
Which of the following would not be a step in your c. 10%
management? d. 15%
a. Intravenous antibiotics
b. Debridement of necrotic tissue 37–29. When is mastitis most likely to occur?
c. Establishment of adequate analgesia prior to a. Postpartum day 1
debridement b. Postpartum day 5
d. Intravenous antibiotics, debridement of necrotic c. 3–4 weeks postpartum
tissue in the operating room, and then immediate d. 6–9 months postpartum
closure of the laceration
37–30. What percentage of women with mastitis develop an
37–25. What is the case-fatality rate of toxic shock abscess?
syndrome? a. 1%
a. 1–2% b. 3%
b. 5–6% c. 10%
c. 10–15% d. 15%
d. 20–25%
37–31. Which of the following is not expected in cases of
37–26. A 20-year-old G1P1 presents 3 days postpartum mastitis?
after a vaginal delivery for fever, headache, nausea/ a. Fever
vomiting, and lower abdominal pain. The patient’s
b. Chills
boyfriend reports that she got sick very quickly and
is “not making sense when she talks.” On exam, the c. Breast firmness
patient has severe abdominal pain and foul-smelling d. Symptoms in both breasts
lochia. She is hypotensive and tachycardic. She
appears extremely ill. Which of the following is the
most likely diagnosis?
a. Listeriosis
b. Pyelonephritis
c. Gastroenteritis
d. Toxic shock syndrome
CHAPTER 37
number answer cited Header cited
37–1 a p. 666 Puerperal Fever
37–2 d p. 666 Puerperal Fever
37–3 a p. 667 Puerperal Fever
37–4 a p. 667 Uterine Infection
37–5 a p. 667 Uterine Infection
37–6 c p. 668 Uterine Infection
37–7 a p. 668 Uterine Infection
37–8 d p. 668 Uterine Infection
37–9 b p. 668 Choice of Antimicrobials
37–10 d p. 669 Table 37-2 Antimicrobial regimens for pelvic infections
following cesarean delivery
37–11 a p. 669 Perioperative Prophylaxis
37–12 d p. 669–670 Perioperative Prophylaxis
37–13 c p. 670 Complications of Uterine and Pelvic Infections
37–14 c p. 670–671 Vacuum-Assisted Wound Closure
37–15 d p. 671 Abdominal Incisional Infections
37–16 a p. 671 Necrotizing Fasciitis
37–17 d p. 671 Adnexal Abscesses and Peritonitis
37–18 c p. 672 Adnexal Abscesses and Peritonitis
37–19 c p. 672 Figure 37-3
37–20 d p. 672 Parametrial Phlegmon
37–21 c p. 673 Septic Pelvic Thrombophlebitis
37–22 b p. 673 Septic Pelvic Thrombophlebitis
37–23 d p. 674 Perineal Infections
37–24 d p. 675 Early Repair of Infected Episiotomy
37–25 c p. 675 Toxic Shock Syndrome
37–26 d p. 675 Toxic Shock Syndrome
37–27 c p. 675 Toxic Shock Syndrome
37–28 b p. 675 Breast Infections
37–29 c p. 675 Breast Infections
37–30 c p. 675 Breast Infections
37–31 d p. 675 Breast Infections
37–32 c p. 676 Breast Abscess
CHAPTER 38
Contraception
38–1. According to the World Health Organization, with 38–5. Which of the following statements properly describe
no contraceptive use, a sexually active woman has the intrauterine device?
what risk for pregnancy over a year? a. Mirena is a levonorgestrel-eluting device, and it is
a. 65% approved for 5 years of use following insertion.
b. 70% b. Liletta contains 52 mg of levonorgestrel, and it is
c. 75% approved for 5 years of use following insertion.
d. 85% c. Skyla is the largest of the intrauterine devices, and
it is approved for 3 years of use following insertion.
38–2. In contrast to the answer to Question 38–1, typical d. ParaGard is not considered a “chemically active”
use of combination oral contraceptive pills by a intrauterine device, and it is approved for 7 years
sexually active woman has what risk for pregnancy of use following insertion.
over the first year of use?
a. 3% 38–6. Contraceptive efficacy with the method pictured below
is not believed to result from which of the following
b. 9%
mechanisms?
c. 13%
d. 17%
38–7. Which of the following statements properly 38–10. Ms. Thomas from Question 38–8 undergoes an
characterizes expulsion of an intrauterine device? ultrasound with the findings shown below. Which of
a. The cumulative expulsion rate after 3 years is 20%. the following is the next best clinical step?
CHAPTER 38
b. Expulsion is most common in the final years of
approved use.
c. If a woman is unable to palpate the trailing
strings, she should be evaluated.
d. All of the above
38–12. Which of the following is true regarding the intra- 38–14. Which of the following is true regarding the
uterine device–related complication pictured here? risk for infection associated with an intrauterine
device (IUD)?
SECTION 10
38–16. Which of the following is most accurate regarding 38–20. Which of the following is true regarding placement
pregnancy with an intrauterine device (IUD) in situ? of the progestin implant?
a. Risk for spontaneous abortion is approximately a. For women who are certain they are not pregnant,
CHAPTER 38
four times as high if the device is left in situ. insertion can occur at any time with no need for
b. Retention of an IUD during pregnancy increases backup contraception.
risk for fetal malformation and preterm delivery. b. For women transitioning from combination oral
c. After viability has been reached, data is definitive contraceptives, it is inserted on what would be
that the IUD should be left in place even if the first day of a new pack.
strings are visible. c. Insertion can occur prior to discharge home
d. Evidence of pelvic infection during a pregnancy following miscarriage, abortion, or delivery with
with a retained IUD should be treated with no impairment of lactation.
antibiotics and uterine evacuation. d. For women not currently using hormonal
contraception, it is ideally inserted 5 days prior
38–17. Which of the following is true regarding insertion of to expected menses onset with contraception
an intrauterine device (IUD)? established within 24 hours.
a. Implanon cannot be easily identified for removal
with ultrasound and has lost approval by the 38–21. Which of the following is a suspected method of
Food and Drug Administration. efficacy for all progestin-only contraceptive methods?
b. For placement unrelated to pregnancy, the only a. Endometrial atrophy
recommended time is near the end of menstruation b. Inhibition of ovulation
as it is easier, and pregnancy is excluded. c. Thickening of cervical mucus
c. With immediate postabortion or postdelivery d. All of the above
placement, fewer women will receive and retain
their IUD compared to those scheduled to return 38–22 Ms. Hubbard is a 28-year-old G6P5 who is seeing
for traditionally timed placement. you for her postpartum exam. She had a repeat
d. All of the above cesarean delivery 2 weeks ago and desires reversible
contraception. Her pregnancy was complicated
38–18. Which of the following is true regarding progestin by gestational diabetes requiring insulin. She also
implants? takes medication for depression. She asks about
a. Implanon cannot be easily identified for removal Nexplanon. Which of the following is an appropriate
with ultrasound and has lost approval by the statement as part of her contraceptive counseling?
Food and Drug Administration. a. Nexplanon will further increase your risk for overt
b. Nexplanon provides 3 years of contraception by diabetes due to weight gain and increased insulin
releasing etonogestrel, and it is implanted in the resistance.
upper, medial surface of the arm. b. Nexplanon is a good choice, but may increase
c. The first implantable progestin contraceptive was your depression, so I would like you to call me if
Norplant, which released etonogestrel from six you notice changes.
subdermal rods, but it is no longer manufactured. c. According to the United Stated Medical Eligibility
d. All of the above Criteria, depression is a contraindication to pro-
gestin only contraceptive methods.
38–19. Ms. Bradley calls your office a few weeks after d. Nexplanon will reduce your breast milk
insertion of her Nexplanon and reports that she has production, but if you choose this method
had numbness and tingling in her arm. Which of the we should insert it today before ovulation can
following characterizes the most likely approximate resume.
region of her symptoms?
a. Anterior and posterior portions of her shoulder
b. Lateral surface of her forearm extending inferiorly
to the base of her thumb
c. Anterior and posterior portions of her 5th digit
and the medial half of her 4th digit
d. Medial aspect of her forearm extending superiorly
to the anterior surface of her upper arm
38–23. Which of the following statements most accurately 38–27. Which of the following statements is true regarding
describes the injectable progestin contraceptive depot patient-specific factors that impact choice of
medroxyprogesterone acetate (DMPA)? combination oral contraceptive (COC) pill?
SECTION 10
a. When injection is given within 5 days of the a. Ongoing irregular bleeding may improve from
onset of menses, a backup method is needed for using a pill with a higher estrogen dose.
7 days. b. Women with significant menstrual symptoms
b. Although irregular bleeding leads to 25% of users may benefit from an extended cycle formulation.
discontinuing within the first year, amenorrhea c. Patients on heparin should have their potassium
develops in 80% of users after 1 year. monitored in the first month of using a COC
c. For women desiring only a brief period of with the progestin drospirenone.
contraception, DMPA is not a good choice due d. All of the above
to prolonged anovulation after discontinuation.
d. All of the above 38–28. Which of the following is not true regarding the
relationship of combination oral contraceptive
38–24. Combination oral contraceptive (COC) pills differ (COC) pills and thrombotic events?
from progestin-only “mini-pills” in which of the a. Etonogestrel-containing COCs are linked to
following ways? greater risk.
a. COC pills primarily inhibit ovulation, and progestin b. Highest risk is in women with a co-morbid
only pills do not reliably inhibit ovulation. thrombophilia.
b. COC pills are contraindicated in women with c. COCs are contraindicated for women over
breast cancer, but progestin only pills are not. 35 years of age who smoke.
c. When one COC dose is missed, a barrier method d. Thrombotic events are increased with COC use
should be used for 7 days. Whereas, when one in the 4 weeks prior to a major operation and in
progestin pill is taken >4 hours late, a barrier the first weeks postpartum.
method needs to be used for 48 hours.
d. All of the above 38–29. Relative risk is increased for which of the following
cancers with combination oral contraceptive (COC)
38–25. The combination oral contraceptive pill’s mechanism pill use?
of action is best described by which of the following a. Ovarian
statements? b. Cervical
a. The estrogen component suppresses luteinizing c. Endometrial
hormone. d. Hepatocellular carcinoma
b. Overall mechanism is stimulation of the hypo
thalamic gonadotropin-releasing factors.
c. Pituitary secretion of follicle-stimulating hormone
and luteinizing hormone are blocked, and
ovulation is inhibited
d. All of the above
38–30. With use of the contraceptive method pictured, how 38–32. In counseling a woman who is considering the
frequently should it be replaced? contraceptive method pictured here, which of the
following is not true?
CHAPTER 38
a. Daily
b. Weekly a. The ring must be carefully inserted such that it
remains in an oblique lie around the cervix.
c. Every 9 days
b. The ring can be removed during intercourse but
d. Every 3 weeks should be replaced within 3 hours of removal.
38–31. Ms. Adams is a 35-year-old multigravida who is c. The ring is more forgiving than combination oral
seeing you for her annual exam. She has hypertension contraceptive pills, as it remains efficacious if left
that is well controlled with a single agent. You are in place for a fourth week.
encouraged that three times per week she is still d. The ring is placed within 5 days of menses onset
doing the water aerobics you recommended last year. and stays in place for 3 weeks with removal
Her blood pressure is 130/78 mm Hg and weight is prompting withdrawal bleeding.
92 kg, which is down from 98 kg at last year’s visit.
She inquires about starting the contraceptive patch 38–33. Contraceptive efficacy of the male latex condom is
for her birth control method. Which characteristic enhanced by which of the following?
below is not a reason she cannot use the patch? a. Reservoir tip
a. Patient weight may reduce efficacy b. Oil-based spermicide
b. Regular immersion in water limits patch efficacy c. Concurrent female condom use
c. A 35-year-old with hypertension presents excess d. All of the above
cardiac risk
d. All of the above make this a poor choice 38–34. Ms. Burns is a 26-year-old G1P1001 with a partner
who has a history of herpes. She desires contracep-
tion, and advice on how she can reduce the risk for
getting genital herpes. In addition to counseling her
regarding symptoms and lesions, you discuss which
of the following?
a. The female condom is not an option if she is
allergic to latex.
b. The female condom has a lower contraceptive
failure rate than the male condom.
c. The female condom is a single-use barrier method
that provides both contraception and protection
against sexually transmitted diseases.
d. All of the above should be considered.
38–35. Which of the following is not true regarding spermi- 38–38. Which of the following is false regarding emergency
cide use? contraception?
a. Spermicide enhances the contraceptive efficacy of a. The major mechanism of action with all methods
SECTION 10
CHAPTER 38
number answer cited Header cited
38–1 d p. 680 Table 38-1
38–2 b p. 680 Table 38-1
38–3 c p. 681 Introduction
38–4 d p. 681 Introduction
38–5 a p. 682 Intrauterine Devices
38–6 a p. 683 Intrauterine Devices
38–7 c p. 683 Intrauterine Devices
38–8 a p. 683 Intrauterine Devices
38–9 c p. 683 Intrauterine Devices
38–10 c p. 683 Intrauterine Devices
38–11 d p. 683 Intrauterine Devices
38–12 a p. 683 Intrauterine Devices
38–13 d p. 683 Intrauterine Devices
38–14 d p. 684 Intrauterine Devices
38–15 a p. 684 Intrauterine Devices
38–16 d p. 684 Intrauterine Devices
38–17 a p. 685 Intrauterine Devices
38–18 b p. 687 Progestin Implants
38–19 d p. 688 Progestin Implants
38–20 c p. 688 Progestin Implants
38–21 d p. 689 Progestin-Only Contraceptives
38–22 b p. 689 Progestin-Only Contraceptives
38–23 c p. 693 Injectable Progestin Contraceptives
38–24 a p. 690 Combination Oral Contraceptive Pills
p. 693 Progestin-Only Pills
38–25 c p. 689 Combination Hormonal Contraceptives Mechanism of Action
38–26 b p. 690 Combination Oral Contraceptive Pills
38–27 d p. 690 Combination Oral Contraceptive Pills
38–28 a p. 691 Combination Oral Contraceptive Pills
38–29 b p. 692 Combination Oral Contraceptive Pills
38–30 b p. 692 Transdermal Patch
38–31 b p. 692 Transdermal Patch
38–32 a p. 693 Transvaginal Ring
38–33 a p. 694 Barrier Methods
38–34 c p. 680 Table 38-1
p. 694 Barrier Methods
38–35 b p. 695 Spermicides
38–36 a p. 695 Fertility Awareness-Based Methods
38–37 d p. 680 Table 38-1
p. 683 Contraceptive Action
p. 689 Actions and Side Effects
p. 695 Fertility Awareness-Based Methods
38–38 c p. 696 Emergency Contraception
CHAPTER 39
Sterilization
39–1. Among women using contraception, what percentage 39–6. A 36-year-old G3P2 desires permanent sterilization
use sterilization? after delivery. She has read recently that the entire
a. 10% fallopian tube should be removed when perform-
ing a tubal ligation. What is the rationale for this
b. 15%
recommendation?
c. 20%
a. Decrease cancer risks
d. 33%
b. Decrease the failure rate
39–2. A 28-year-old G4P3 presents for prenatal care at c. Decrease bleeding complications
8 weeks’ gestation and reports she does not want any d. Decrease the risk for an ectopic pregnancy
more children. She asks for a postpartum bilateral
tubal ligation. All except which of the following are 39–7. What method of tubal ligation is displayed below?
accurate and should be included in your counseling?
a. It is permanent
b. There are failure rates
c. It can be reversed without consequence
d. An explanation of alternative options for
contraception
39–9. You are seeing a 33-year-old G3P2 for prenatal care. 39–12. Which of the following is true regarding identification
She had two prior vaginal deliveries and desires a of the structure being ligated?
bilateral tubal ligation after this pregnancy. She a. The distal fimbria must be seen prior to ligation.
CHAPTER 39
anticipates having another vaginal delivery and
b. Common reason for failure of sterilization is
wants to know what type of incision she will have
ligation of the wrong structure.
for her tubal ligation. Which of the answer choices
below depicts the best incision for a puerperal tubal c. The midportion of the fallopian tube can be
ligation? confused with that of the round ligament.
d. All of the above
a. A
b. B
c. C
d. D
39–16. What is the cumulative failure rate for tubal 39–20. A 27-year-old multigravida tells you at her 28-week
sterilization? prenatal care visit that she desires a tubal ligation
a. 0.5% after delivery. For which of the following reasons do
SECTION 10
a. Zinc
b. Copper
c. Progesterone
d. Nickel and titanium alloy
39–25. When after Essure placement should confirmation 39–29. Which of the following is a reason why pregnancy
with a hysterosalpingography be performed? after a vasectomy may occur?
a. 3 weeks a. Recanalization
CHAPTER 39
b. 6 weeks b. Incomplete occlusion
c. 12 weeks c. Unprotected intercourse too soon
d. 16 weeks d. All of the above
39–26. During a vasectomy, which of the following structures 39–30. Pregnancy rates after vasectomy reversal increase
is ligated? with all except which of the following?
a. Epididymis a. Microsurgical technique
b. Spermatic cord b. Younger female partner age
c. Efferent ductile c. Longer duration from vasectomy to reversal
d. Ductus deferens d. Normal sperm quality during reversal procedure
39–27. Compared with tubal ligation, which of the 39–31. Compared to a traditional tubal ligation, which of
following is an advantage of vasectomy? the following is increased with a salpingectomy?
a. Less invasive a. Risk for bleeding
b. Easily reversed b. Risk for postoperative ileus
c. Immediately effective c. Risk for adhesion formation
d. Does not require confirmation d. All of the above
39–28. How long does it take for sperm to be completely 39–32. Why should the bladder be emptied prior to a
removed from the reproductive tract? puerperal tubal ligation?
a. 1 week a. Avoid bladder injury
b. 4 weeks b. Prevent postop urinary retention
c. 12 weeks c. Prevent the fundus from dropping to the pubic
d. 16 weeks symphysis
d. All of the above